Question

Prove Corollary 4.22: A set of real numbers E is closed and bounded if and only...

Prove Corollary 4.22: A set of real numbers E is closed and bounded if and only if every infinite subset of E has a point of accumulation that belongs to E.

Use Theorem 4.21: [Bolzano-Weierstrass Property] A set of real numbers is closed and bounded if and only if every sequence of points chosen from the set has a subsequence that converges to a point that belongs to E.

Must use Theorem 4.21 to prove Corollary 4.22 and there should be no mention of closed and bounded in the proof. The proof should start with,

[E closed and bounded] iff [E has the BW Property]

Homework Answers

Know the answer?
Your Answer:

Post as a guest

Your Name:

What's your source?

Earn Coins

Coins can be redeemed for fabulous gifts.

Not the answer you're looking for?
Ask your own homework help question
Similar Questions
Show that if a set of real numbers E has the Heine-Borel property then it is...
Show that if a set of real numbers E has the Heine-Borel property then it is closed and bounded.
1) Prove: Any real number is an accumulation point of the set of rational number. 2)...
1) Prove: Any real number is an accumulation point of the set of rational number. 2) prove: if A ⊆ B and A,B are bounded then supA ≤ supB . 3) Give counterexample: For two sequences {an} and {bn}, if {anbn} converges then both sequences are convergent.
Using the completeness axiom, show that every nonempty set E of real numbers that is bounded...
Using the completeness axiom, show that every nonempty set E of real numbers that is bounded below has a greatest lower bound (i.e., inf E exists and is a real number).
Prove: Let S be a bounded set of real numbers and let a > 0. Define...
Prove: Let S be a bounded set of real numbers and let a > 0. Define aS = {as : s ∈ S}. Show that inf(aS) = a*inf(S).
1. (a) Let S be a nonempty set of real numbers that is bounded above. Prove...
1. (a) Let S be a nonempty set of real numbers that is bounded above. Prove that if u and v are both least upper bounds of S, then u = v. (b) Let a > 0 be a real number. Define S := {1 − a n : n ∈ N}. Prove that if epsilon > 0, then there is an element x ∈ S such that x > 1−epsilon.
Let E⊆R (R: The set of all real numbers) Prove that E is sequentially compact if...
Let E⊆R (R: The set of all real numbers) Prove that E is sequentially compact if and only if E is compact
Prove that the set of real numbers of the form e^n,n= 0,=+-1,+-2,... is countable.
Prove that the set of real numbers of the form e^n,n= 0,=+-1,+-2,... is countable.
Using the real numbers as the model, which of the following are true and which are...
Using the real numbers as the model, which of the following are true and which are not true? a. Some finite point set has a limit point. b. Any infinite number set M has a limit point. c. Any subset of (a, b) has a limit point. d. All limit points of a given set belong to the set. --------------------------------------------------------------------------------------------- Definition. A point p is said to be a limit point of a point set M if and only if...